Hi,
I've looked at lot of explanations (I have starter so I don't have access to 7Sage explanation) and I'm still not able to understand this one. I figured out the conditionals during the test, but I still don't fully understand how to approach/ ...
As you know, I'm a big LSAT nerd. I love LSAT and I love LSATters. 7sage is a huge part of my life. Some of my most important contacts and favorite buddies have come through this forum. So, I'm looking for a way to give back ... ...
Admin note: Please review the forum rules.
@"J.Y. Ping" said:
3. Do not post LSAT questions, any copyrighted content, or links to content that infringe on copyright. Not a good way to take the ...
I don't understand the correct answer for this one at all. Can someone breakdown why all the wrong ones are correct and D is correct? Here is my breakdown:
This question just doesn't seem to click for me any which way I look at it... I don't see how answer choice (A) is the correct answer. If someone has any insight as to how to better approach this question, be my guest! Thanks!
http://7sage.com/ ...
http://7sage.com/lsat_explanations/lsat-june-2007-section-3-question-15/
Hey, could anyone please help me understand how answer choice B does not weaken the argument? if there were more people with more than 6 months treatment responding then the ...
Can somebody please point me in the directions of games that are similar to this one? Thanks!
https://7sage.com/lsat_explanations/lsat-77-section-3-game-4/
https://7sage.com/lsat_explanations/lsat-56-section-3-question-21
I understand Jon's explanation on this question, but I was tricked by the question's word choice "revival" in the answer choice (D).
Doesn't "revival" imply that the ballroom ...
Even though the question stem doesn't explicitly say "which one of the following could be a *COMPLETE AND ACCURATE* list of people selected" like what most of other questions do, do I still need to automatically assume that the list needs to be complete? ...
Damn, PT52 has some pretty tough LR sections, and even after a retake, I missed many of the same question again (like this one). I don't see how answer A weakens the argument nor how B doesn't.
I'm redoing some questions that I marked when I first went through the ciriculum, and I came across this tricky one. I fully see why answer D is correct, but I can't figure out what makes B incorrect. Doesn't answer B deny an alternate cause?
https://7sage.com/lsat_explanations/lsat-53-section-3-question-13
I can't see the difference between the flaw in B and the the flaw in the incorrect answer choices....
I made this comment on the video 5 months ago, but I still would like to get an answer or opinion on this: So for MSS/MBT questions containing other people’s opinion and author’s opinion, we should always choose answer that states author’s opinion because ...
https://7sage.com/lsat_explanations/lsat-41-section-3-question-23
I understand the reasoning in the video, that just because X is preceded by Y, it doesn’t mean that Y is the necessary condition.
Okay, so I really debated between "C" and "D" (the former apparently being a crappy choice), and ended up selecting "C" only because the term "approving" in "D" seemed ...
Can some of you awesome people weigh in on statistical reasoning as it's used in the LSAT? For example, PT 18 S2 Q4. Specifically, things like probability or likelihood trip me up. Any resources or explanations would be appreciated! Thanks!
I tried this game for the second time after a five months period, I got all questions right just under the target time but I spent nearly two minutes looking at the conditional rules to find some kind of deep inference which was a huge waste of time. Is it ...
Most Strongly Supported with Sage Corey Janson
Friday 5/20 9pm ET
Back by popular demand (y'all are practically beating the door down on this one). Corey gives us round 2 of his most excellent MSS intensive this Friday!
I'm so lost on this question. It asks which statement would most appropriately continue the discussion at the end of the passage. I ruled out A, because of tone, and selected E. However, apparently A is the right answer. Is anyone able to explain why? I ...
In the link below, can someone explain number 8? The comments don't provide a clear answer, and I have no idea what it means. It makes no sense how dull means "placing demands on readers" to me
How I interpreted the question was the minimum number of salaries would be 8 not 7. The reason for this was that K would always be first, thus leading the other 8 workers being placed in the remaining ...